Which one of the following could be a complete and accurate list of the messages left on the answering machine, from ...

Lenny on April 6, 2021

Doesn't P also have to be in the 6 with H L and T?

If you have T 3 times then you need 2 more, H and L are given and it has to be P as a 6th because you cannot input G or F there. Please correct my incompetence!

Reply
Create a free account to read and take part in forum discussions.

Already have an account? log in

Lenny on April 6, 2021

If you have T 3 times you need 3 more*